Xem bài viết đơn
Old 13-08-2012, 01:47 PM   #39
Snow Bell
+Thành Viên Danh Dự+
 
Tham gia ngày: Apr 2012
Đến từ: Heaven
Bài gởi: 579
Thanks: 10
Thanked 513 Times in 283 Posts
Trích:
Nguyên văn bởi quykhtn View Post
tanggo nên dùng kí hiệu $ \sum $ để lời giải gọn hơn

Bài toán 15 Cho các số thực dương $ a,b,c $ thỏa mãn $ abc=1 $.Chứng minh rằng $$ 1) \ \ \ \ \sqrt[3]{\dfrac{a^3+b^3}{2}}+\sqrt[3]{\dfrac{b^3+c^3}{2}}+\sqrt[3]{\dfrac{c^3+a^3}{2}}+6 \leq 3(a+b+c) $$
Bài toán này có thể chứng minh bằng cách sử dụng đánh giá sau:
$$ \sqrt[3]{\frac{a^3+b^3}{2}} \le \frac{a^2+b^2}{a+b} $$
Chú ý rằng:
$$ \frac{a^2+b^2}{a+b}=a+b-\frac{2ab}{a+b} $$
Như vậy ta phải chứng minh:
$$ 2\left[\frac{ab}{a+b}+\frac{bc}{b+c}+\frac{ca}{c+a} \right]+a+b+c \ge 6 $$
Áp dụng bất đẳng thức AM-GM với giả thiết $ abc=1 $,ta có ngay:
$$ \frac{2ab}{a+b}+\frac{a+b}{2}+ \frac{2bc}{b+c}+ \frac{b+c}{2}+\frac{2ca}{c+a}+\frac{c+a}{2} \ge 6 $$
Vậy ta có ìiều phải chứng minh.Đẳng thức xảy ra khi $ a=b=c=1 $
[RIGHT][I][B]Nguồn: MathScope.ORG[/B][/I][/RIGHT]
 
Snow Bell is offline   Trả Lời Với Trích Dẫn
The Following 2 Users Say Thank You to Snow Bell For This Useful Post:
K56khtn (16-08-2012), quykhtn (22-08-2012)
 
[page compression: 8.97 k/10.05 k (10.78%)]